ChaseDream

标题: LSAT-22-2-12 [打印本页]

作者: rt316    时间: 2003-9-11 06:42
标题: LSAT-22-2-12
22-2-12(D)-觉得应该是B
One year ago a local government initiated an antismoking

advertising campaign in local newspapers which it financed by

imposing a tax on cigarettes of 20 cents per pack. One year later

the number of people in the locality who smoke cigarettes had

declined by 3 percent. Clearly what was said in the

advertisements had an effect although a small one on the number

of people in the locality who smoke cigarettes.
Which one of the following, if true, most helps to strengthen

argument?
(A) Residents of the locality have not increased their use of

other tobacco products such as snuff and chewing tobacco since

the campaign went into effect
(B) A substantial number of cigarette smokers in the locality who

did not quit smoking during the campaign now smoke less than they

did before it began
(C) Admissions to the local hospital for chronic respiratory

ailments were down by 15 percent one year after the campaign

began
(D) Merchants in the locality responded to the local tax by

reducing the price at which they sold cigarettes by 20 cents per

pack
(E) Smokers in the locality had incomes that on average were 25

percent lower than those of nonsmokers

作者: mindfree    时间: 2003-9-11 07:40
B is a weaken answer.

You need to read the question more carefully. The conclusion is "Clearly what was said in the advertisements had an effect although a small one on the number of people in the locality who smoke cigarettes".

B will weaken the conclusion by saying that it is not that the number of smokers dropped, but that people smoked less.

D is the right answer. Basically it indicates that the price of cigarettes did not increase due to the imposed tax. In another word, as the price did not go up, the drop in number of smokers is not the result of price change.

Hope I answered your question.





欢迎光临 ChaseDream (https://forum.chasedream.com/) Powered by Discuz! X3.3